Added topic tags to some Y&F-12 problems
authorW. Trevor King <wking@drexel.edu>
Tue, 25 Aug 2009 06:53:32 +0000 (02:53 -0400)
committerW. Trevor King <wking@drexel.edu>
Thu, 17 Sep 2009 16:50:56 +0000 (12:50 -0400)
13 files changed:
latex/problems/Young_and_Freedman_12/problem27.22.tex
latex/problems/Young_and_Freedman_12/problem27.30.tex
latex/problems/Young_and_Freedman_12/problem27.35.tex
latex/problems/Young_and_Freedman_12/problem27.39.tex
latex/problems/Young_and_Freedman_12/problem27.64.tex
latex/problems/Young_and_Freedman_12/problem27.68.tex
latex/problems/Young_and_Freedman_12/problem27.73.tex
latex/problems/Young_and_Freedman_12/problem28.12.tex
latex/problems/Young_and_Freedman_12/problem28.18.tex
latex/problems/Young_and_Freedman_12/problem28.23.tex
latex/problems/Young_and_Freedman_12/problem28.30.tex
latex/problems/Young_and_Freedman_12/problem28.60.tex
latex/problems/Young_and_Freedman_12/problem28.62.tex

index 789d09048c35a5619571d1a0715be010fd5fff27..43cce5645aec9ea1dc2fde7d46ccc0b674f1122a 100644 (file)
@@ -1,4 +1,4 @@
-\begin{problem*}{27.22}
+\begin{problem*}{27.22} % cyclotrons
 In an experiment with cosmic rays, a verticle beam of particles that
 have charge of magnitude $3e$ and mass $12$ times the proton mass
 enters a uniform horizontal magnetic field of $0.250\U{T}$ and is bent
index 3825683a8ca5ae00be0685cc65683d7b03a5e9e3..5435f8f58a1dee6e93655cacac735e1e287650ca 100644 (file)
@@ -1,4 +1,4 @@
-\begin{problem*}{27.30}
+\begin{problem*}{27.30} % Lorentz force
 A particle with initial velocity $\vect{v}_0=5.85\E{3}\U{m/s}\jhat$
 enters a region of uniform electric and magnetic fields.  The magnetic
 field in the region is $\vect{B}=-(1.35\U{T})\khat$.  Calculate the
index 4c33eedf505a2b57dd1bd47a7c4bcdaf12eedf7f..6b9ee4b2b69f4078802538ba57232d29997e34d9 100644 (file)
@@ -1,4 +1,4 @@
-\begin{problem*}{27.35}
+\begin{problem*}{27.35} % magnetic force on wires
 A long wire carrying $4.50\U{A}$ of current makes two $90\dg$ bends,
 as shown in Fig.~27.49.  The bent part of the wire passes through a
 uniform $0.240\U{T}$ magnetic field direceted as shown in the figure
index 7268a975c1a77b86fc1b03bf1bc2b4797b6fe88e..259d73e5a91d2aec76361021a26d741033be947d 100644 (file)
@@ -1,4 +1,4 @@
-\begin{problem*}{27.39}
+\begin{problem*}{27.39} % rail guns
 A thin, $50.0\U{cm}$ long metal bar with mass $750\U{g}$ rests on, but
 is not attached to, two metallic supports in a uniform $0.450\U{T}$
 magnetic field, as shown in Fig.~27.51.  A battery and a
index 0f7361ad80860c6dab2d021315968abe305fc1bf..add47136ed7ede833b4932671654f510bd028be9 100644 (file)
@@ -1,4 +1,4 @@
-\begin{problem*}{27.64}
+\begin{problem*}{27.64} % magnetic force on charges
 A particle of charge $q>0$ is moving at speed $v$ in the
 $+z$-direction through a region of uniform magnetic field \vect{B}.
 The magnetic force on the particle is $\vect{F}=F_0(3\ihat+4\jhat)$,
index d89c89f11e04ea4739672b1b42acd141eaec1f4f..b1cbaeb5dc544652d19f93f54a5bf71629a68cff 100644 (file)
@@ -1,4 +1,4 @@
-\begin{problem*}{27.68}
+\begin{problem*}{27.68} % rail guns
 A $3.00\U{N}$ metal bar, $1.50\U{m}$ long and having a resistance of
 $10.0\U{\Ohm}$, rests horizontally on conducting wires connecting it
 to the circuit shown in Fig.~27.62.  The bar is in a uniform,
index a6e118ef20fd6975b31a0483906100d790dcafc0..ec47cc2c1f281dd14bb21b1a311163c8420e1fb4 100644 (file)
@@ -1,4 +1,4 @@
-\begin{problem*}{27.73}
+\begin{problem*}{27.73} % magnetic force on wires
 A long wire carrying a $6.00\U{A}$ current reverses direction by means
 of two right-angle bends, as shown in Fig.~27.64.  The part of the
 wire where the bend occurs is in a magnetic field of $0.666\U{T}$
index 30df69accb0e1508024910145ce331af40f204e5..3d293dad4d8ed55c535b5250767310b1ee328846 100644 (file)
@@ -1,7 +1,7 @@
 \newcommand{\dB}{d\vect{B}}
 \newcommand{\dl}{d\vect{l}}
 
-\begin{problem*}{28.12}
+\begin{problem*}{28.12} % Biot-Savart law, magnetic field from wires
 Two parallel wires are $5.00\U{cm}$ apart and carry currents in
 opposite directions, as shown in Fig.~28.37.  Find the magnitude and
 direction of the magnetic field at point $P$ due to the two
index 0fc786aa5a95f6eddbc0a176904912ca73a0f8b4..fea23f956cd1c4813c4a76bc87a89596a5c1c259 100644 (file)
@@ -1,4 +1,4 @@
-\begin{problem*}{28.18}
+\begin{problem*}{28.18} % magnetic field from wires
 Two long, straight wires, one above the other, are seperated by a
 distance $2a$ and are parallel to the $x$-axis.  Let the $+y$-axis be
 in the plane of the wires in the direction from the lower wire to the
index cb598017d481db65257bf69b05ab1b9d5beb1a61..91b5cfe3e0cc0dde8b8b689a3c2aa69d4e8dc8f1 100644 (file)
@@ -1,4 +1,4 @@
-\begin{problem*}{28.23}
+\begin{problem*}{28.23} % magnetic field from wires
 Four long, parallel power lines each carry $100\U{A}$ currents.  A
 cross-sectional diagram of these lines if a square, $20.0\U{cm}$ on
 each side.  For each of the three cases shown in Fig.~28.41, calculate
index b7177fa64536e315afc692807da4bc2f1847b4ee..b81ab536805b33714e9a942efecde30c4bc16b78 100644 (file)
@@ -2,7 +2,7 @@
 %\newcommand{\dB}{d\vect{B}}
 %\newcommand{\dl}{d\vect{l}}
 
-\begin{problem*}{28.30}
+\begin{problem*}{28.30} % Biot-Savart law, magnetic field from wires
 Calculate the magnitude and direction of the magnetic field at point
 $P$ due to the current in the semicircular section of wire shown in
 Fig.~28.46.  (\emph{Hint:} Does the current in the long, straight
index bbda9f7ebe7cf8177a44e62c26461eb9aee4a1d4..69880c988ba05c71d76b20572fbae26666034ef7 100644 (file)
@@ -1,4 +1,4 @@
-\begin{problem*}{28.60}
+\begin{problem*}{28.60} % magnetic field from wires
 Figure~28.54 shows an end view of two long, parallel wires
 perpendicular to the $xy$-plane, each carrying a current $I$ but in
 opposite directions.  \Part{a} Copy the diagram, and draw vectors to
index 4ad478996c88d13713ddd4d4001b9ebd93c4ab84..76f366c2877bef7d3acfa5a1fdb50cdd10e550a9 100644 (file)
@@ -1,4 +1,4 @@
-\begin{problem*}{28.62}
+\begin{problem*}{28.62} % magnetic force on wires, magnetic field from wires
 A pair of long, rigid metal rods, each of length $L$, lie parallel to
 each other on a perfectly smooth table.  Their ends are connected by
 identical, very light conducting springs of force constant $k$